LSAT and Law School Admissions Forum

Get expert LSAT preparation and law school admissions advice from PowerScore Test Preparation.

 Rosaline
  • Posts: 17
  • Joined: Apr 29, 2018
|
#48938
Hello,

I'm struggling with why answer choice C is correct and D is wrong? I also momentarily considered B. Could someone please explain why C is correct and B and D are wrong? Thank you!
 Adam Tyson
PowerScore Staff
  • PowerScore Staff
  • Posts: 5153
  • Joined: Apr 14, 2011
|
#49007
Thanks for the question, Rosaline! Here's the judge's argument from the stimulus, broken down step by step:

1. The suspect ran from the police
2. Running from the police does not, by itself, create a reasonable suspicion of a crime
3. Evidence gathered as a result of an illegal chase is inadmissible
4. The weapon gathered as a result of the chase was therefore inadmissible

There's a missing link in this chain of reasoning, and that link is between steps 2 and 3 above, where I boldfaced two concepts. How did we get from this chase not raising a reasonable suspicion to the chase being illegal? In order to strengthen the argument, we need to bridge that gap. We do that by prephrasing "if there is no reasonable suspicion for chasing someone, then that chase is illegal". Answer choice C gives us the contrapositive of that prephrase - if a chase is legal, there was reasonable suspicion of a crime.

Answer choice B fails to close that gap in the reasoning because it makes no mention of "reasonable suspicion". It doesn't give us any reason to accept the reasoning that this chase was illegal. Maybe the person fleeing was committing a crime, so their flight was illegal, so maybe the chase was okay? Or maybe they weren't committing a crime, so their flight was legal, so...maybe the chase was illegal or maybe not? We just can't be sure here whether the chase was legal or illegal, not without knowing more.

Answer choice D tells us that running isn't a crime, but does that mean chasing in this case is illegal? Again, this makes no connection between "reasonable suspicion" and "illegal chase", and so it doesn't help the argument any. Okay, running wasn't a crime, but could it still be legal for the police to give chase? I can't tell from this claim, so this doesn't help.

Watch for gaps in arguments all over the LSAT, as that is where the answers will often lie. Gaps can be described in Flaw in the Reasoning answers, Assumption answers, Strengthen or Justify answers, even Weaken answers (which would take advantage of the gap - here, that would be something like "a chase can still be legal even if the person's flight doesn't raise a reasonable suspicion of a criminal act"). Mind the gap!
 Rosaline
  • Posts: 17
  • Joined: Apr 29, 2018
|
#49192
Thanks. This was very helpful. I get it now.
 lsatfighter
  • Posts: 26
  • Joined: Sep 26, 2018
|
#62374
I found this question to be the hardest one in this LR section, but I see the gap in the argument and why C is right.

Just to make sure: Is A wrong because it only strengthens a premise of the judge? Is B wrong because it uses the words "legally FLEE", which is out of scope as the argument talks about legal/illegal CHASE? Is D wrong because it simply restates a premise of the judge? And is E wrong because it's a mistaken negation of the gap?

Furthermore, I can see that the first 2 sentences of the stimulus were just fluff. Can you give me some advice on how to get past all the fluff and clutter UNDER TIMED CONDITIONS in order to better understand an argument? Do you have any advice on how to better understand the structure of an argument, especially noticing gaps (like the one in this question) under timed conditions? Furthermore, can you please explain to me why A is wrong?
 Robert Carroll
PowerScore Staff
  • PowerScore Staff
  • Posts: 1787
  • Joined: Dec 06, 2013
|
#62409
fighter,

Answer choice (A) doesn't even strengthen a premise. When the judge said that "by itself, flight does not create reasonable suspicion," the judge is not committed to something like "...but other factors combined with flight might create such suspicion." We don't know why the judge used the phrase "by itself" - possibly because the judge thought other factors might matter, possibly not. If, in fact, flight from the police never creates reasonable suspicion, by itself or otherwise, that would be great for the judge's argument! So the qualification in answer choice (A) is at best irrelevant to the judge's argument, and at worst bad for it. It provides no help.

Answer choice (B) won't help the argument because it mentions a necessary condition for flight to be legal. This never tells me that flight IS legal - it can only tell me when flight ISN'T legal - if the necessary condition is lacking. If the necessary condition is present, I cannot conclude anything about whether flight was legal - thinking I could infer legality from the necessary condition's truth would be a Mistaken Reversal. So this answer does nothing.

Answer choice (D) doesn't really restate a premise - it says whether flight should be considered a criminal act, not whether it gives rise to reasonable suspicion of a criminal act (the latter is what the judge was saying). It's wrong because it fails to close the gap.

On a question late in a Logical Reasoning section, you should expect some distracting extraneous information. This is one reason you want to save time on early questions, so you can devote more time to later questions, where the test makers may be "hiding the ball" a bit more. In this specific case, the first two sentences just present facts. There's not much to understand there, because there's not even an attempt to link those facts into an argument yet. The third sentence, though, presents itself as an argument, so I'd perk up if I were reading it - any time an argument is made, it's going to be relevant! Since the rest of the stimulus presents the argument of the judge, this is where I'd devote all my time.

Here, the judge laid out the premises in a logical order. This actually makes it easier to identify gaps - because each premise is supposed to build off the last, any time new information is introduced there's a gap. If anything the judge said here didn't follow from what he/she already said, there's a gap. In this case, "illegal chase" is an entirely new concept that's not implied by the previous premises.

Robert Carroll

Get the most out of your LSAT Prep Plus subscription.

Analyze and track your performance with our Testing and Analytics Package.